Đến nội dung

HaiDangPham nội dung

Có 314 mục bởi HaiDangPham (Tìm giới hạn từ 07-06-2020)



Sắp theo                Sắp xếp  

#745141 CMR: $KF, LE$ cắt nhau tại $S$ trên đường tròn ngoại tiếp...

Đã gửi bởi HaiDangPham on 23-05-2024 - 18:20 trong Hình học

Chứng minh của bạn @Hoang Long Le thật sự hay và gọn. Hai tam giác SFB và SEC đồng dạng là điều mình thấy ngay lúc đầu tìm kiếm lời giải nhưng lại không nghĩ ra được cách chứng minh. Không ngờ là vẫn có thể chứng minh điều đó mà không cần hình phụ. 

 

Bài toán trên còn có thể mở rộng thêm như sau: 

d) PQ, KL và BC đồng quy. 

e) Gọi T là điểm đối xứng với A qua tâm O của đường tròn ngoại tiếp tam giác ABC. Khi đó ba điểm S, I, T thẳng hàng. (Ý này có thể mở rộng hơn nữa: bốn điểm S, I, J, T thẳng hàng trong đó J là tâm đường tròn ngoại tiếp tứ giác PQCB). 




#745112 CMR: $KF, LE$ cắt nhau tại $S$ trên đường tròn ngoại tiếp...

Đã gửi bởi HaiDangPham on 21-05-2024 - 17:53 trong Hình học

Tóm lược. Bài toán trên thực sự khó. Các hình phụ quan trọng nhất gồm điểm J, tâm đường tròn ngoại tiếp tứ giác $PQCB$, và đường kính $MN$ của đường tròn $(O)$ ngoại tiếp tam giác $ABC$.

Toàn bộ chứng minh của bài toán gồm 3 bổ đề và 13 mệnh đề, được chia thành ba phần, hướng đến mục đích chính là chứng minh $S$ thuộc tâm đường tròn $(O)$. Phần I, gồm 3 bổ đề. Phần II, gồm các mệnh đề từ 1 đến 8. Phần III, gồm 5 mệnh đề còn lại. Các Mệnh đề 9, 10, 11 lần lượt vận dụng ba bổ để đề thiết lập nên các kết quả quan trọng nhất chuẩn bị cho hai mệnh đề cuối 12 và 13. 

Việc chia chứng minh thành các mệnh đề như trên nhằm mục đích giúp độc giả dễ theo nắm bắt được các bước chính trong suy luận. Ngoài ra mỗi mệnh đề tự nó cũng là một bài toán hay, khi tách riêng ra ta sẽ ý thức được rõ hơn cái hay của mỗi bài toán đó.

 

Về phương pháp giải. Nói thêm về phương pháp tư duy của mình khi giải toán hình. Điều quan trọng nhất: tập trung vào giả thiết bài toán. Việc suy luận ngược từ kết luận thường không hiệu quả trong những bài hình khó. Sự tập trung vào giả thiết này gồm hai phương diện thứ nhất là liên hệ với kiến thức đã biết, và thứ hai là dựa vào những quan hệ đã có ta tiếp tục tạo ra các quan hệ mới.

1) Từ giả thiết ta liên hệ với những tính chất quen thuộc đã biết. Chẳng hạn, nếu giả thiết cho I là tâm đường tròn nội tiếp tam giác ABC thì ta nhớ ngay ra rằng nếu gọi M là giao điểm của AI và đường tròn ngoại tiếp tam giác ABC thì M là điểm chính giữa cung BC.  

2) Tạo ra hình phụ. Không có một nguyên tắc nhất định nào, nhưng trong kinh nghiệm của mình thì thường làm như sau: 

(i) Tạo ra thêm đường thẳng mới, và đường tròn mới. 

(ii) Tìm giao điểm của các đường thẳng và đường tròn đã có. 

(iii) Quan sát và dự đoán các tính chất đặc biệt. Có trung điểm nào mới, tia phân giác nào mới không? Có ba điểm nào thẳng hàng không? Có ba đường thẳng nào đồng quy không? Có hai đường thẳng nào vuông góc hay song song không? Có đường thẳng và đường tròn nào tiếp xúc nhau hay hai đường tròn tiếp xúc nhau không? Có tứ giác nào nội tiếp không? Có hai tam giác nào bằng nhau hay đồng dạng không?...Trong bước này thì sự hỗ trợ của GeoGebra là thực sự hữu ích. Với những bài khó ta phải liên tục thử nghiệm các hình phụ mới và đưa ra dự đoán, nhờ có GeoGebra mà ta nhanh chóng kiểm tra được dự đoán của ta là đúng hay sai. 

Sau khi có được những tính chất mới mà ta đánh giá là hay và gần sát với mục tiêu thì ta mới bắt đầu tìm cách chứng minh bằng lập luận dự đoán của ta là đúng. Nếu chưa tìm ra cách chứng minh thì ta cứ để đó và tìm thêm các tính chất mới. Sau rất nhiều tìm tòi các hình phụ như thế ta sẽ tới một thời điểm tìm ra được một chuỗi các tính chất dẫn thẳng đến đích. Và công việc cuối cùng chỉ còn là hoàn tất chứng minh cho các tính chất khó vẫn còn bỏ ngỏ ở giai đoạn tìm kiếm. 

 

Phần I. 

Bổ đề
Cho tam giác $ABC$ đồng dạng tam giác $A’B’C’$ theo tỉ số đồng dạng $k>1$. $O$ là tâm đường tròn ngoại tiếp tam giác $ABC$, $O’$ là tâm đường tròn ngoại tiếp tam giác $A’B’C’$. Khi đó nếu hai tam giác đã cho có các cặp cạnh tương ứng song song thì bốn đường thẳng $AA’, BB’, CC’, OO’$ đồng quy.

Bđ 1.png

Chứng minh. Gọi $X$ là giao điểm của $AA’$ và $BB’$.

Tam giác $XAB$ có $A’B’ \parallel AB$ nên theo Định lý Thales ta có $\frac{XA’}{XA}=\frac{A’B’}{AB}$. Mà $\frac{A’B’}{AB}=\frac{A’C’}{AC}$ ($\Delta{ABC} \sim \Delta{A’B’C’}$) nên $\frac{XA’}{XA}=\frac{A’C’}{AC}$. Mặt khác $AC \parallel A’C’$ nên $\angle{XA’C’}=\angle{XAC}$ (hai góc đồng vị). Do đó $\Delta{XA’C’} \sim \Delta{XAC}$ dẫn tới $\angle{A’XC’}=\angle{AXC}$. Do đó tia $XC’$ trùng với tia $XC$, nói cách khác $CC’$ đi qua $X$.

Tam giác $OAC$ cân tại $O$,  tam giác $O’A’C’$ cân tại $O’$. Lại có $\angle{AOC}=2\angle{ABC}$, $\angle{A’OC’}=2\angle{A’B’C’}$, $\angle{ABC}=\angle{A’B’C’}$ nên $\angle{AOC=A’OC’}$. Do đó $\Delta{OAC} \sim \Delta{O’A’C’}$. Thêm nữa $AC \parallel A’C’$ nên không mấy khó khăn ta suy ra $OA  \parallel O’A’$, $OC \parallel O’C’$.

Ta có $\frac{XA’}{XA}=\frac{A’C’}{AC}$, mà $\frac{O’A’}{OA}=\frac{A’C’}{AC}$ (do $\Delta{OAC} \sim \Delta{O’A’C’}$) nên $\frac{XA’}{XA}=\frac{O’A’}{OA}$. Ta có $OA \parallel O’A’$ nên $\angle{XA’O’}=\angle{XAO}$ (hai góc đồng vị), do đó $\Delta{XA’O’} \sim \Delta{XAO}$, suy ra $\angle{A’XO’}=\angle{AXO}$. Do đó $OO’$ đi qua $X$.

Vậy bốn đường thẳng $AA’, BB’, CC’, OO’$ đồng quy tại $X$.

 

Bổ đề
Cho tứ giác $ABCD$ ngoại tiếp đường tròn $(I)$. Đường tròn $(I)$ tiếp xúc $AB, BC, CD, DA$ lần lượt tại $M, N, P, Q$. Khi đó bốn đường thẳng $AC, BD, MP, NQ$ đồng quy.

Bđ2.png

Chứng minh. Ở đây ta chỉ chứng minh trường hợp tứ giác $ABCD$ có hai cặp cạnh đối cắt nhau. Trường hợp tứ giác $ABCD$ có một hoặc cả hai cặp cạnh đối song song xin dành cho độc giả.

Gọi $E$ là giao điểm $AD$ và $BC$, $F$ là giao điểm $AB$ và $CD$. $K_1$ là giao điểm $AC$ và $MP$, $K_2$ là giao điểm $AC$ và $NQ$.

Áp dụng Định lý Menelaus cho tam giác $ACF$ và ba điểm thẳng hàng $K_2, P, M$ ta có $\frac{K_2A}{K_2C}.\frac{PC}{PF}.\frac{MF}{MA}=1$ $(1)$

Áp dụng Định lý Menelaus cho tam giác $ACE$ và ba điểm thẳng hàng $K_1, N, Q$ ta có $\frac{K_1A}{K_1C}.\frac{NC}{NE}.\frac{QE}{QA}=1.$ $(2)$

Theo tính chất của hai tiếp tuyến cắt nhau ta có $PF=MF, NE=QE, QA=MA, PC=NC$, khi đó từ (1) và (2) ta suy ra $\frac{K_1A}{K_1C}=\frac{K_2A}{K_2C}$. Chứng tỏ $K_1, K_2$ trùng nhau. Vậy $AC, MP, NQ$ đồng quy.

Chứng minh tương tự $BD, MP, NQ$ đồng quy. Ta có điều phải chứng minh.

 

Bổ đề
Cho tam giác $ABC$ đồng dạng tam giác $A’B’C’$. Điểm $M$ thuộc $BC$, $M’$ thuộc $B’C’$ sao cho $MB:MC=M’B’:M’C’$. Khi đó $\Delta{AMB} \sim \Delta{A’M’B’}$, $\Delta{AMC} \sim \Delta{A’M’C’}$.

Bđ3.png

Bổ đề này chứng minh tương đối đơn giản xin dành cho bạn đọc.

 

Phần II.

Mđ1-8.png

Mệnh đề

PQCB là tứ giác nội tiếp.

Ta có $AI^2=AP.AB$ và $AI^2=AE.AC$ do $AI$ là tiếp tuyến chung của đường tròn $(K)$ và đường tròn $(L)$. Vì vậy $AP.AB=AQ.AC$, suy ra $PQCB$ là tứ giác nội tiếp.

 

Gọi $M$ là giao điểm của $AI$ và đường tròn $(O)$.

 

Mệnh đề
M là tâm đường tròn ngoại tiếp tam giác BIC.

Dễ thấy $\angle{IBM}=\angle{IBC}+\angle{CBM}$ và $\angle{BIM}=\angle{IBA}+\angle{IAB}$.  Mà $\angle{IBC}=\angle{IBA}$ và $\angle{CBM}=\angle{IAB}$ nên $\angle{IBM}=\angle{BIM}$. Ta có tam giác $IBM$ cân tại $M$. Tương tự tam giác $ICM$ cân tại $M$. Vậy $MB=MI=MC$, tức là $M$ là tâm đường tròn ngoại tiếp tam giác $BIC$.

 

Mệnh đề
$MB$ là tiếp tuyến của đường tròn $(K)$; $MC$ là tiếp tuyến của đường tròn $(L)$

 

Do $MB=MI, KB=KI$ nên $\Delta{BKM}=\Delta{IKM}$, suy ra $\angle {KBM}=\angle{KIM}=90^{\circ}$. Vậy $MB$ là tiếp tuyến của đường tròn $(K)$. Tương tự ta có $MC$ là tiếp tuyến của đường tròn $(L)$.

 

Mệnh đề
$\angle{BKI}=\angle{BPQ}$ và $\angle{CLI}=\angle{PQC}$.
 

Theo giả thiết và Theorem, ta có $MB, MI$ đều là tiếp tuyến của đường tròn $(K)$, vì vậy $\angle{BKI}=180^{\circ}-\angle{IMB}$. Mà $\angle{BPQ}=180^{\circ}-\angle{BCQ}$ (vì tứ giác $PQCB$ nội tiếp theo Theorem và $\angle{BCQ}=\angle{IMB}$ (tứ giác $ACMB$ nội tiếp đường tròn), nên $\angle{BKI}=\angle{BPQ}$. Tương tự $\angle{CLI}=\angle{PQC}$.

 

Mệnh đề
Đường tròn $(I)$ nội tiếp tứ giác $PQCB$.
 

Ta có $\angle{IPB}=\frac{\angle{BKI}}{2}$ và $\angle{BKI}=\angle{BPQ}$ (Theorem) vì vậy $\angle{IPB}=\frac{\angle{BPQ}}{2}$, tức $PI$ là tia phân giác của góc $BPQ$. Từ đây dễ thấy $PQ$ tiếp xúc với đường tròn $(I)$, và do đó đường tròn $(I)$ nội tiếp tứ giác $PQCB$.

 

Gọi $N$ là giao điểm của $KB$ và $LC$.

 

Mệnh đề
$MN$ là đường kính của đường tròn $(O)$; $MN$ là đường trung trực của $BC$.
 

Theo Theorem, $MB$ là tiếp tuyến của đường tròn $(K)$, $MC$ là tiếp tuyến của đường tròn $(L)$ nên $\angle{MBN}=\angle{MCN}=90^{\circ}$. Do đó $MN$ là đường kính của đường tròn ngoại tiếp tam giác $MBC$, nói cách khác $MN$ là đường kính của đường tròn $(O)$.

Ta có $MB=MC$ do $AM$ là tia phân giác góc $BAC$. Mà $OB=OC$ nên $OM$ là đường trung trực của $BC$. Mà $O$ thuộc $MN$, do $MN$ là đường kính của đường tròn (O) nên $MN$ cũng là đường trung trực của $BC$.

 

Gọi $J$ là tâm đường tròn ngoại tiếp tứ giác $PQCB$.

Mệnh đề
Tứ giác KJLN nội tiếp.
 

Ta có $JK$ là đường trung trực của $PB$ và $JL$ là đường trung trực của $QC$. Do đó $JK \perp AB$, $JL \perp AC$, dẫn tới $\angle{KJL}+\angle{BAC}=180^{\circ}$. Theo Theorem, ta lại có $N$ thuộc đường tròn $(O)$ nên $\angle{KNL}=\angle{BAC}$. Suy ra $\angle{KJL}+\angle{KNL}=180^{\circ}$. Vậy tứ giác $KJLN$ nội tiếp.

 

Mệnh đề
$J$ nằm trên đường kính $MN$ của đường tròn $(O)$ và $J$ là tâm đường tròn ngoại tiếp tam giác $KML$.
 

Vì $J$ là tâm đường tròn ngoại tiếp tứ giác $PQCB$ nên $J$ thuộc đường trung trực của $BC$, mà $MN$ là đường trung trực của $BC$ (Theorem) nên $J$ thuộc $MN$. 

Do đó $NJ$ là tia phân giác góc $KNL$, mà tứ giác $JKNL$ nội tiếp  (Theorem) nên $JK=JL$. 

Theo Theorem, $MK$ là tiếp tuyến của đường tròn $(K)$, $ML$ là tiếp tuyến của đường tròn $(L)$ nên $MK$ là tia phân giác góc $IDB$, $ML$ là tia phân giác góc $CDI$. Do đó $\angle{KML}=\frac{\angle{BMC}}{2}$.

Tứ giác $KJLN$ nội tiếp nên $\angle{KJL}=\angle{BMC}=180^{\circ}-\angle{BAC}$. Do đó $\angle{KML}=\frac{\angle{KJL}}{2}$. 

Mà theo chứng minh trên $JK=JL$ nên dễ thấy $J$ là tâm đường tròn ngoại tiếp tam giác $KML$.

 

Phần III.

Gọi $D$ là điểm tiếp xúc của $BC$ và đường tròn $(I)$.

Mđ9.png

Mệnh đề
S, I, J thẳng hàng; S, D, M thẳng hàng.
 

Hai tam giác $DEF$ và $MLK$ đồng dạng và có các cặp cạnh tương ứng song song ($DF \parallel MK, DE \parallel ML, EF \parallel KL$). $I$ là tâm đường tròn ngoại tiếp tam giác $DEF$, $J$ là tâm đường tròn ngoại tiếp tam giác $MLK$ (Theorem). Áp dụng Theorem ta có $MD, KF, LE, IJ$ đồng quy tại $S$. Vậy $S, I, J$ thẳng hàng; $S, D, M$ thẳng hàng.

 

Mđ11.png

 

Mệnh đề
DI là tia phân giác của góc KDL.
 

Ta có $M$ là tâm đường tròn ngoại tiếp tam giác $IBC$ (Theorem). Gọi $D*$ là giao điểm của ID và đường tròn (M). Tiếp tuyến tại $D^{*}$ của đường tròn $(M)$ cắt $KB$ và $LC$ lần lượt tại $B^{*}$ và $C^{*}$. Ta có ngay đường tròn $(M)$ nội tiếp tứ giác $KLC^{*}B^{*}$.

Dễ thấy $\angle{D^{*}MC}=\angle{DIE} (=2\angle{DIC})$. Mà $DIEC$ và $CMD^{*}C^{*}$ là các tứ giác nội tiếp nên $\angle{LC^{*}B^{*}}=180^{\circ}-\angle{D^{*}MC}$ và $\angle{QCB}=180^{\circ}-\angle{DIE}$. Do đó $\angle{LC^{*}B^{*}}=\angle{QCB}$.

Mặt khác, theo Theorem ta có $\angle{LKB^{*}}=\angle{BPQ}$. Do đó $\angle{LKB^{*}}+\angle{LC^{*}B^{*}}=\angle{BPQ}+\angle{QCB}$. Mà theo Theorem, tứ giác $PQCB$ nội tiếp nên $\angle{BPQ}+\angle{QCB}=180^{\circ}$. Do đó $\angle{LKB^{*}}+\angle{LC^{*}B^{*}}=180^{\circ}$. Vậy $B^{*}KLC^{*}$ là tứ giác nội tiếp. Suy ra $\angle{B^{*}KC^{*}}=\angle{B^{*}LC^{*}}$.

Đường tròn $(M)$ tiếp xúc với $KL, LC^{*}, B^{*}C^{*}, KB^{*}$ lần lượt tại $I, C, D^{*}, B$. Trong đó $D$ là giao điểm của $BC$ và $D^{*}I$. Theo Theorem ta có $KC^{*}$ cắt $LB^{*}$ tại $D$. Vì vậy $\angle{BKD}=\angle{CLD}$.

Từ đây dễ thấy $\Delta{BKD} \sim \Delta{CLD}$ (g.g). Do đó $\angle{KDB}=\angle{LDC}$. Mà $ID \perp BC$ nên $\angle{IDK}=\angle{IDL}$. Vậy $DI$ là tia phân giác của góc  $KDL$.

 

Mệnh đề
Tam giác JIK đồng dạng tam giác MDB; tam giác JIL đồng dạng tam giác MDC.
 

Tam giác $JKL$ cân tại $J$, tam giác $MBC$ cân tại $M$, và $\angle{KJL}=\angle{BMC}$ nên $\Delta{JKL} \sim \Delta{MBC}$. Theo Theorem ta có $DI$ là tia phân giác của góc $KDL$ nên $\frac{IK}{IL}=\frac{DK}{DL}$, mà $\frac{DK}{DL}=\frac{DB}{DC}$ (do $\Delta{BKD} \sim \Delta{CLD}$), nên $\frac{IK}{IL}=\frac{DB}{DC}$. Do đó theo Theorem  $\Delta{JIL} \sim \Delta{MDC}$ và $\Delta{JIK} \sim \Delta{JIL}$.

Mđ12-13.png

Mệnh đề
$S$ thuộc đường tròn ngoại tiếp tứ giác $JKNL$.
 

Theo Theorem, ta có $ \Delta{JIL} \sim \Delta{MDC}$, suy ra $\angle{JIL}=\angle{MDC}$.

Mặt khác do đường tròn $(K)$ và đường tròn $(L)$ cùng tiếp xúc với $AI$ tại $I$ nên $KL \perp AM$ tại $I$. Mà theo Theorem, $J$ thuộc $MN$ là đường trung trực của $BC$ nên $JM \perp BC$, dẫn tới $\angle{JIM}=\angle{JMD}$.

Theo Theorem ba điểm $S, D, M$ thẳng hàng và ba điểm $S, I, J$ thẳng hàng nên $\angle{JIM}=\angle{JMS}$ và $\angle{IJM}=\angle{MJS}$. Suy ra $\Delta{JIM} \sim \Delta{JMS}$ (g.g). Suy ra $JM^2=JI.JS$. Mà $JK=JM$ (Theorem) nên $JK^2=JI.JS$.

Hai tam giác $JIK$ và $JKS$ chung góc $J$. Lại từ đẳng thức $JK^2=JI.JS$ ta có $\frac{JK}{JS}=\frac{JI}{JK}$ nên hai tam giác này đồng dạng (c.g.c). Do đó $\angle{JSK}=\angle{JKI}$.

Theo Theorem, $JK=JL$ nên $\angle{JKI}=\angle{JLK}$. Do đó $\angle{JSK}=\angle{JLK}$.  Suy ra $S$ thuộc đường tròn ngoại tiếp tam giác $JKL$. Mà tứ giác $JKNL$ nội tiếp (Theorem) nên $S$ thuộc đường tròn ngoại tiếp tứ giác $JKNL$.

 

Mệnh đề
S thuộc đường tròn (O).
 

Do $S$ thuộc đường tròn ngoại tiếp tứ giác $JKNL$ (Mệnh đề 12) nên $\angle{SNB}=\angle{SJK}$.

Mà theo chứng minh ở Theorem ta có $\Delta{JKI} \sim \Delta{MBD}$ nên $\angle{SJK}=\angle{SMB}$. Do đó $\angle{SNB}=\angle{SMB}$. Vậy $SNMB$ nội tiếp đường tròn. Mà $MN$ là đường kính đường tròn $(O)$ (Theorem)nên $S$ thuộc đường tròn $(O)$. 




#745111 Chứng minh đường thẳng OI song song với đường thẳng EF

Đã gửi bởi HaiDangPham on 21-05-2024 - 15:49 trong Hình học

Mình nghĩ là không nên dùng từ đồng viên nó kiểu không có từ nào chỉ í, chắc là bạn học ai đó nên dùng từ nội tiếp sẽ dễ hiểu hơn

 

"Đồng viên" dùng cho đối tượng là 4 điểm (một tập hợp các điểm), còn "nội tiếp" dùng cho đối tượng là tứ giác (một hình). Ta có thể nói "4 điểm cùng thuộc một đường tròn" hoặc "4 điểm đồng viên". Còn khi muốn chỉ tới đối tượng là một tứ giác (có trật tự các đỉnh cụ thể) mà 4 đỉnh tứ giác đó thuộc một đường tròn thì ta nói "tứ giác nội tiếp đường tròn". Nói cho dễ hiểu, một "tứ giác nội tiếp" là một tứ giác có "4 đỉnh đồng viên".

 

Hai thuật ngữ "đồng viên" và "nội tiếp" trên hướng tới các đối tượng khác nhau nên không thể đặt ra vấn đề chọn cái này hay bỏ cái kia kiểu thay thế được. Không có cái gọi là "4 điểm nội tiếp" hay "tứ giác đồng viên". Nếu có sự thay thế ở đây thì chỉ là sự thay thế giữa hai cách diễn đạt là "4 điểm đồng viên" hoặc "4 điểm cùng thuộc một đường tròn". "Đồng viên" có lẽ là thuật ngữ đã quá cũ, trong sách giáo khoa hiện nay không còn dùng nữa. 




#743215 9 cách chứng minh định lý ba đường cao đồng quy trong tam giác

Đã gửi bởi HaiDangPham on 24-01-2024 - 21:23 trong Hình học

bạn có file pdf phần này mình xin được không ạ

 

Rất tiếc, mình soạn trực tiếp trên diễn đàn nên không có pdf. 




#743069 [Topic] Đại số trung học cơ sở

Đã gửi bởi HaiDangPham on 17-01-2024 - 14:06 trong Đại số

Bài 15. Cho số thực $a$ thỏa $a^3-a-1=0$, tính giá trị của biểu thức $B=a\sqrt{2a^6-4a^4+4a^2+3a}-\sqrt{2a^2+3a+2}.$ 

 

Từ điều kiện $a^3-a-1=0$ ta suy ra

(i) $a^3=a+1$

(ii) $a^4=a^2+a$

(iii) $a^6=a^2+2a+1$. 

Do đó

$2a^6-4a^4+4a^2+3a$

$=2(a^2+2a+1)-4(a^2+a)+4a^2+3a$

$=2a^2+3a+2$. 

Suy ra 

$B=(a-1)\sqrt{2a^2+3a+2}$

$=\sqrt{(a-1)^2(2a^2+3a+2)}$

$=\sqrt{2a^4-a^3-2a^2-a+2}$

$=\sqrt{2(a^2+a)-(a+1)-2a^2-a+2}$

$=1.$

Vậy $B=1$. 




#743068 [Topic] Đại số trung học cơ sở

Đã gửi bởi HaiDangPham on 17-01-2024 - 12:02 trong Đại số

Bài 14. Cho biểu thức $f(x)=(2x^3-21x+2023)^{2024}$.Tính giá trị của biểu thức $f(x)$ tại $x=\sqrt[3]{7+\sqrt{\frac{49}{8}}}+\sqrt[3]{7-\sqrt{\frac{49}{8}}}$

 

Ta có $x^3=14+3.\sqrt[3]{7^2-\left(\sqrt{\frac{49}{8}}\right)^2}.x=14+\frac{21}{2}.x$, suy ra $2x^3-21x=28$. 

Do đó $f(x)=(28+2023)^{2024}=2051^{2024}$. 




#743059 [Topic] Đại số trung học cơ sở

Đã gửi bởi HaiDangPham on 16-01-2024 - 21:49 trong Đại số

BÀI 11. Vì $x=1+\sqrt[3]{2}$  nên $(x-1)^3=2$ hay $x^3=3x^2-3x+3$, dẫn tới $x^4=3x^3-3x^2+3x$. 

Suy ra $P=x^4-5x^3+9x^2-12x+6$

$=(3x^3-3x^2+3x)-5x^3+9x^2-12x+6$

$=-2x^3+6x^2-9x+6$

$=-2(3x^2-3x+3)+6x^2-9x+6$

$=-3x$

Vậy $P=-3(1+\sqrt[3]{2})=(-3)+\sqrt[3]{-54}$.

Suy ra $a=-3, b=-54$, và ta có $a-2b=(-3)-2.(-54)=105$.




#743057 [Topic] Đại số trung học cơ sở

Đã gửi bởi HaiDangPham on 16-01-2024 - 21:37 trong Đại số

BÀI 16. a) Đặt $ax^3=by^3=cz^3=k^3$.

Khi đó $\sqrt[3]{ax^2+by^2+cz^2}=\sqrt[3]{ax^3.\frac{1}{x}+by^3.\frac{1}{y}+cz^3.\frac{1}{z}}=\sqrt[3]{k^3.(\frac{1}{x}+\frac{1}{y}+\frac{1}{z})}=k$. 

Mặt khác 

$\sqrt[3]{a}+\sqrt[3]{b}+\sqrt[3]{c}=\sqrt[3]{\frac{k^3}{x^3}}+\sqrt[3]{\frac{k^3}{y^3}}+\sqrt[3]{\frac{k^3}{z^3}}=k. (\frac{1}{x}+\frac{1}{y}+\frac{1}{z})=k$

Ta suy ra điều phải chứng minh. 




#743052 [Topic] Đại số trung học cơ sở

Đã gửi bởi HaiDangPham on 16-01-2024 - 18:40 trong Đại số

Bài 11. Đặt $u=\sqrt[3]{a+\frac{a+1}{3}\sqrt{\frac{8a-1}{3}}}$ và $v=\sqrt[3]{a-\frac{a+1}{3}\sqrt{\frac{8a-1}{3}}}$. 

Trước hết, dễ thấy $u^3+v^3=2a$. 

Tiếp theo ta có

$u.v=\sqrt[3]{(a+\frac{a+1}{3}\sqrt{\frac{8a-1}{3}})(a-\frac{a+1}{3}\sqrt{\frac{8a-1}{3}})}$

$=\sqrt[3]{a^2-\frac{(a+1)^2(8a-1)}{27}}$

$=\sqrt[3]{\frac{-(2a-1)^3}{27}}$

$=\frac{-(2a-1)}{3}$. 

Do đó

$x^3 = u^3+v^3+3uv(u+v)$

$=2a+3.\frac{-(2a-1)}{3}.x$

$=2a-(2a-1)x$

Suy ra $x^3+(2a-1)x-2a=0$ hay $(x-1)(x^2+x+2a)=0$. (*) 

Xét trường hợp $a=\frac{1}{8}$ khi đó thay vào công thức tính x ban đầu ta có ngay $x=\sqrt[3]{\frac{1}{8}}+\sqrt[3]{\frac{1}{8}}=1$. 

Trường hợp $a > \frac{1}{8}$ ta có $x^2+x+2a > x^2+x+\frac{1}{4}=(x+\frac{1}{2})^2 \geq 0$. Vì vậy từ phương trình (*) ta có $x=1$. 

Tóm lại $x=1$ với mọi giá trị của $a \geq \frac{1}{8} $, hay $x$ là số tự nhiên với mọi $a \geq \frac{1}{8}$. 

 




#743044 [Topic] Đại số trung học cơ sở

Đã gửi bởi HaiDangPham on 16-01-2024 - 10:59 trong Đại số

Bài 10.

Ta có $\frac{1}{\sqrt[3]{2}+\sqrt[3]{4}+3}=\frac{1}{(\sqrt[3]{2}+1)(\sqrt[3]{4}+1)}$

$=\frac{\sqrt[3]{4}-\sqrt[3]{2}+1}{ (\sqrt[3]{2}+1)(\sqrt[3]{4}-\sqrt[3]{2}+1)}. \frac{\sqrt[3]{16}-\sqrt[3]{4}+1}{ (\sqrt[3]{4}+1)(\sqrt[3]{16}-\sqrt[3]{4}+1)}.$

$=\frac{\sqrt[3]{4}-\sqrt[3]{2}+1}{3}.\frac{\sqrt[3]{16}-\sqrt[3]{4}+1}{5}$

$=\frac{7-\sqrt[3]{2}-2\sqrt[3]{4}}{15}$




#742541 $ab(a + 1) + bc(b + 1) + ca(c + 1) \ge 2$

Đã gửi bởi HaiDangPham on 17-12-2023 - 07:51 trong Bất đẳng thức và cực trị

Cho các số thực a, b, c thỏa mãn: $0 \le a, b, c \le 1$ và $a + b + c \ge 2$

Chứng minh rằng $ab(a + 1) + bc(b + 1) + ca(c + 1) \ge 2$.

 

Xét $a=0$, khi đó VT=$bc(b+1)$. Với điều kiện $0 \leq b, c \leq 1$ ta suy ra VT$\leq 2$.

Bất đẳng thức cần chứng minh không đúng. 

---

Viết xong mới thấy nhận xét không chuẩn. Xin lỗi tác giả ạ. 

Vì khi $b, c \leq 1$ mà $b+c \geq 2$ thì ta phải có $b=c=1$. Khi đó BĐT cần chứng minh xảy ra dấu đẳng thức. Không có gì sai ạ. 




#742534 $\frac{xy}{z}+\frac{yz}{x...

Đã gửi bởi HaiDangPham on 16-12-2023 - 17:52 trong Bất đẳng thức và cực trị

BÀI TOÁN 8. Với $a, b, c,  m, n, p, x, y, z$ là các số thực dương chứng minh rằng $$(a^3+b^3+c^3) (m^3+n^3+p^3) (x^3+y^3+z^3) \geq (amx+bny+cpz)^3$$

Đây là Bất đẳng thức Holder cho 3 dãy. Diễn đàn đã có chứng minh tại đây, tuy nhiên thì bạn nào chưa biết bất đẳng thức này có thể thử tự tìm cách giải. Gợi ý: sử dụng AM-GM. 

 

BÀI TOÁN 9. Với $a, b, c$ là các số thực dương chứng minh rằng 

a) $\frac{a}{\sqrt{a^2+8bc}}+\frac{b}{\sqrt{b^2+8ac}}+\frac{c}{\sqrt{c^2+8ab}} \geq 1$

b) $\frac{a}{\sqrt{2b^2+2c^2-a^2}}+\frac{b}{\sqrt{2a^2+2c^2-b^2}}+\frac{c}{\sqrt{2a^2+2b^2-c^2}} \geq \sqrt{3}$




#742517 $\frac{xy}{z}+\frac{yz}{x...

Đã gửi bởi HaiDangPham on 15-12-2023 - 13:07 trong Bất đẳng thức và cực trị

Hay $S\ge\frac{(2+\sqrt{3}+\sqrt{5})^2}{2}-12$. Do đó $Min S =\frac{(2+\sqrt{3}+\sqrt{5})^2}{2}-12$. 

 

Ở đây, ta cần thêm bước chỉ ra giá trị của $a, b, c$ để dấu đẳng thức xảy ra từ đó mới khẳng định được giá trị nhỏ nhất của $S$, cụ thể  $$a=\frac{2-\sqrt{3}+\sqrt{5}}{2},  b=\frac{-2+\sqrt{3}+\sqrt{5}}{2}, c=\frac{2+\sqrt{3}-\sqrt{5}}{2}.$$

Cảm ơn lời giải của bạn hanguyen445.




#742508 $\frac{xy}{z}+\frac{yz}{x...

Đã gửi bởi HaiDangPham on 14-12-2023 - 20:37 trong Bất đẳng thức và cực trị

BÀI TOÁN 7. Tìm giá trị nhỏ nhất của biểu thức $$S=\frac{3a}{b+c}+\frac{4b}{a+c}+\frac{5c}{a+b}.$$




#742448 $\frac{xy}{z}+\frac{yz}{x...

Đã gửi bởi HaiDangPham on 10-12-2023 - 10:32 trong Bất đẳng thức và cực trị

BÀI TOÁN 6. Giả sử $x, y, z \geq 1$ và $\frac{1}{x}+\frac{1}{y}+\frac{1}{z}=2$. Chứng minh $$\sqrt{x+y+z} \geq \sqrt{x-1}+\sqrt{y-1}+\sqrt{z-1}.$$      




#742447 $\frac{xy}{z}+\frac{yz}{x...

Đã gửi bởi HaiDangPham on 10-12-2023 - 10:28 trong Bất đẳng thức và cực trị

BÀI TOÁN 5. Giả sử $ x\geq y \geq z > 0$. Chứng minh $$\frac{x^2y}{z}+ \frac{y^2z}{x}+\frac{z^2x}{y} \geq x^2+y^2+z^2.$$

 

Lời giải khác cho bài toán trên như sau. 

 

Áp dụng bất đẳng thức Cauchy-Schwarz ta có $$\left( \frac{x^2y}{z}+ \frac{y^2z}{x}+\frac{z^2x}{y} \right). \left( \frac{x^2z}{y}+ \frac{y^2x}{z}+\frac{z^2y}{x} \right) \geq (x^2+y^2+z^2)^2.$$ 

Mặt khác vì $x \geq y \geq z$ nên $$ \frac{x^2y}{z}+ \frac{y^2z}{x}+\frac{z^2x}{y} - \frac{x^2z}{y} - \frac{y^2x}{z} - \frac{z^2y}{x}=\frac{(xy+yz+zx)(x-y)(x-z)(y-z)}{xyz} \geq 0.$$ Từ đó ta có điều phải chứng minh. Đẳng thức xảy ra khi $x=y=z$. 




#742340 $\frac{xy}{z}+\frac{yz}{x...

Đã gửi bởi HaiDangPham on 03-12-2023 - 13:49 trong Bất đẳng thức và cực trị

\begin{align*}VT-VP &=x^2(\frac{y}{z}-1)+y^2(\frac{z}{x}-1)+z^2(\frac{x}{y}-1) \\ &=\frac{x^2(y-z)}{z}+\frac{y^2(z-x)}{x}+\frac{z^2(x-y)}{y}\\ &\geq\frac{x^2(y-z)}{x}+\frac{y^2(z-x)}{x}+\frac{z^2(x-y)}{x}\\ &=\frac{x^2(y-z)+y^2(z-x)+z^2(x-y)}{x} \\ &=\frac{(x-y)(x-z)(y-z)}{x}\geq 0 \end{align*}

Do đó $$\frac{x^2y}{z}+\frac{y^2z}{x}+\frac{z^2x}{y}\geq x^2+y^2+z^2 $$

(mà đề bài phải là $x\geq y\geq z>0$ mới đúng chứ)

 

Cảm ơn bạn Duc3290 đã góp ý điều kiện bài toán. Mình đã sửa lại. Lời giải bạn cũng rất hay. 

 

Mình đã viết lại Latex cho chứng minh của bạn. Với những chuỗi đánh giá bất đẳng thức dài, ta nên xuống và căn lề để tránh tràn màn hình. Bạn ấn vào Trả lời để xem câu lệnh cụ thể sau nhé:   

\begin{align*} \end{align*}



#742311 $\frac{xy}{z}+\frac{yz}{x...

Đã gửi bởi HaiDangPham on 01-12-2023 - 18:55 trong Bất đẳng thức và cực trị

BÀI TOÁN 5. Giả sử $ x\geq y \geq z > 0$. Chứng minh $$\frac{x^2y}{z}+ \frac{y^2z}{x}+\frac{z^2x}{y} \geq x^2+y^2+z^2.$$




#742276 $\frac{xy}{z}+\frac{yz}{x...

Đã gửi bởi HaiDangPham on 28-11-2023 - 14:11 trong Bất đẳng thức và cực trị

BÀI TOÁN 4.

Cho các số thực dương $a, b, c$. Chứng minh rằng $$ \frac{a}{(b+c)^2}+\frac{b}{(c+a)^2}+\frac{c}{(a+b)^2} \geq \frac{9}{4(a+b+c)}.$$




#742266 $\frac{xy}{z}+\frac{yz}{x...

Đã gửi bởi HaiDangPham on 27-11-2023 - 18:49 trong Bất đẳng thức và cực trị

Hi vọng có thể tìm được cách giải nữa cho Bài toán 3. 




#742258 $\frac{xy}{z}+\frac{yz}{x...

Đã gửi bởi HaiDangPham on 27-11-2023 - 09:57 trong Bất đẳng thức và cực trị

BÀI TOÁN 3.

Cho các số dương $a, b, c$ thoả mãn điều kiện $a+b+c=3$. Chứng minh bất đẳng thức $$\frac{a}{1+b^2}+\frac{b}{1+c^2}+\frac{c}{1+a^2} \geq \frac{3}{2}.$$




#742243 $\frac{xy}{z}+\frac{yz}{x...

Đã gửi bởi HaiDangPham on 26-11-2023 - 19:47 trong Bất đẳng thức và cực trị

Bài toán 2. Chứng minh rằng với mọi $a, b, c, d$ dương ta luôn có $$16(abc+bcd+cda+dab) \leq (a+b+c+d)^3.$$




#742241 $\frac{xy}{z}+\frac{yz}{x...

Đã gửi bởi HaiDangPham on 26-11-2023 - 19:42 trong Bất đẳng thức và cực trị

Bài này có thể xem như là dạng đổi biến của bất đẳng thức cơ bản $(a+b+c)^2 \geq 3(ab+bc+ca)$.

 

Thật vậy nếu đặt $a=\frac{yz}{x},b=\frac{xz}{y}, c=\frac{xy}{z} $ thì $ab=z^2, bc=x^2, ca=y^2$, cho nên $ab+bc+ca=x^2+y^2+z^2=3$. Vì vậy $$(a+b+c)^2 \geq 3(ab+bc+ca) \Leftrightarrow (a+b+c)^2 \geq 9 \Leftrightarrow a+b+c \geq 3.$$ Đây là điều phải chứng minh. 




#742232 $\frac{xy}{z}+\frac{yz}{x...

Đã gửi bởi HaiDangPham on 26-11-2023 - 13:23 trong Bất đẳng thức và cực trị

Bài toán 1. Cho các số thực dương $x, y, z$ thoả mãn $x^2+y^2+z^2=3$. Chứng minh $$\frac{xy}{z}+\frac{yz}{x}+\frac{zx}{y} \geq 3.$$ 




#742063 Chứng minh rằng $\widehat{BNC} + \widehat{BMC...

Đã gửi bởi HaiDangPham on 08-11-2023 - 13:08 trong Hình học

Cho tam giác $ABC$ nội tiếp đường tròn $(O)$.Các đường cao $BE,CF$ cắt nhau tại $H$. Gọi $M,N$ lần lượt là trung điểm $AH,EF$. Chứng minh rằng $\widehat{BNC} + \widehat{BMC}=180^o$

 

 

Dang-DDTH-2023Thang11Ngay8 copy.jpg

 

Khá thích bài này. Một tính chất thú vị. Chứng minh không khó cho lắm. 

 

LỜI GIẢI. 

 

Gọi $P$ là trung điểm $BC$ và $Q$ là điểm đối xứng của $M$ qua $P$. 

 

* Tam giác $AEH$ vuông tại $E$ có $M$ là trung điểm cạnh huyền $AH$ nên $MA=ME$, từ đó suy ra $\angle AEM=\angle MAE$. Tương tự trong tam giác $BEC$ ta có $\angle PEC=\angle PCE$.

 

Vì $AH \perp BC$ nên $\angle MAE+\angle PCE=90^{\circ}$. 

 

Suy ra $\angle AEM+\angle PEC=90^{\circ}$, dẫn tới $\angle PEM=90^{\circ}.$

 

Vậy $PE \perp EM$. Tương tự ta có $PF \perp MF$.

 

* $M$ là tâm đường tròn ngoại tiếp tứ giác AEHF nên $ME=MF$. Hai tam giác vuông $PEM$ và $PFM$ có chung cạnh huyền $PM$ và có $ME=MF$ nên bằng nhau. Suy ra $PE=PF$. Do đó $PM$ là đường trung trực của $EF$. Chứng tỏ $PM$ vuông góc với $EF$ tại $N$. 

 

*  Vì P đồng thời  là trung điểm $BC$ và $QM$ nên tứ giác $BMCQ$ là hình bình hành. 

 

* Theo hệ thức lượng trong tam giác vuông $PEM$ ta có $PE^2=PM.PN$. Mà $PC=PE$ nên $PC^2=PM.PN$. Dẫn tới tam giác $PNC$ và tam giác $PCM$ đồng dạng (c.g.c). Suy ra $\angle PNC=\angle PCM$.

 

Mặt khác $\angle PCM=\angle PBQ$ (vì $CM \parallel BQ$) nên $\angle PNC=\angle PBQ$. Chú ý rằng $M, N, P$ thẳng hàng và $M, P, Q$ thẳng hàng, suy ra $N, P, Q$ thẳng hàng. Do đó $\angle CNQ=\angle CBQ$. 

 

Vì vậy tứ giác $BNCQ$ là tứ giác nội tiếp. Suy ra $\angle BNC+\angle BQC=180^{\circ}$. Mà $\angle BQC=\angle BMC$ (tứ giác $BMCQ$ là hình bình hành), nên ta có điều phải chứng minh $\angle BMC+\angle BNC=180^{\circ}$.